LSAT and Law School Admissions Forum

Get expert LSAT preparation and law school admissions advice from PowerScore Test Preparation.

 Administrator
PowerScore Staff
  • PowerScore Staff
  • Posts: 8917
  • Joined: Feb 02, 2011
|
#27404
Complete Question Explanation

Cannot Be True—PR. The correct answer choice is (A)

In this question we are asked to find an answer choice containing a principle that is violated by Terry’s reasoning. This means that the correct answer will go directly against the argument that Terry makes, and can therefore be considered as Cannot be True.

Terry concludes that the insurance company is deliberately avoiding paying his claim for the bicycle because he has submitted the claim three times and received no settlement. He admits that human error could be a cause, but the persistence of the error makes him believe that it is deliberate.

Answer choice (A): This is the correct answer choice. This answer states that consumers should NOT attribute dishonesty to a corporation when human error/incompetence might be to blame, which directly contradicts Terry. Remember, Terry does attribute dishonesty to the corporation, despite the admission that human error/incompetence might be the cause of the problem.

Answer choice (B): This answer does not contradict Terry, as Terry did attempt to keep him/herself informed (called the firm repeatedly).

Answer choice (C): Again, this answer does not contradict Terry, as Terry bases the conclusion solely on his/her own experience.

Answer choice (D): The morality of the corporation is never discussed, so this answer cannot be related to Terry’s argument.

Answer choice (E): Terry’s argument is about the cause for the delay in payment (deliberate vs. human error), so an answer choice about public access to information is unrelated to the argument.
 psik
  • Posts: 6
  • Joined: Jul 13, 2015
|
#20540
Stimulus:

Terry: Months ago, I submitted a claim for my stolen bicycle to my insurance company...

Answer A: Consumers should avoid attributing dishonestly to a corproation when the actions of the corproations must instead be exxplained by incompetence.

My question/confusion:

First, I cannot understand why A is the write/best answer: Terry claims he has called repeatedly to ensure this is not some sort of a 'mistake' on the part of the company, therefore he may argue that indeed "I have made sure this is NOT a case of incompetence but deliberate avoidance" --> thus this answer is true and supportive of Terry's argument, or am I making a mistake here by making up stuff for Terry which Terry never claiemd. In other words why is it that A MUST NOT BE TRUE? Is it not that it could be true?

Second, why is D such a bad answer? Compared to A?
D says: In judging the morality of a corporation's behavior, as opposed to that of an indvidual, mitigating circumstnaces are irrelevant.

Thank you
User avatar
 Dave Killoran
PowerScore Staff
  • PowerScore Staff
  • Posts: 5852
  • Joined: Mar 25, 2011
|
#20549
Hi Psik,

Yes, you are reading into Terry's conclusions a bit too much. What we know is that Terry went through all these steps. Did he do it to deliberately prove it wasn't incompetence? No, we don't know that that was what he was attempting to prove, and, in fact he hasn't proven that it's not incompetence. any one of us who have dealt with large companies has found this type of thing, where you have an interaction and then later on they have no idea what you are talking about. what we have here is Terry contacting them a lot, and them initially telling him that they have no record, and then ultimately not sending him his settlement. He draws the conclusion that it's deliberate, but do we know that for sure? No, nothing has been proven.

So, we then get a Cannot question, and they want to know which answer choice contains a principle that is violated. Did Terry violate the reasoning used in (A)? Yes, he definitely did. Answer choice (A) says you should avoid saying it's deliberate when it could be incompetence, and yet Terry said it was deliberate when incompetence was still a possibility.

With (D), I wouldn't say this is violated. The focus is on ignoring mitigating circumstances, and for this answer to be correct, Terry would have to explicitly take into account some mitigating circumstances. Did he do that? No, he didn't, and hence this answer wasn't violated and thus is incorrect.

This type of question is always tough because they way you have to think to solve it feels upside down. And, that's exacerbated here because the answer choices are principles, not more factually-oriented statements. So, when you see this question type, you really have to make sure you are clear on what would constitute a violation of the principle. If you can lock in on that, it makes it easier.

Please let me know if this helps. Thanks!

Get the most out of your LSAT Prep Plus subscription.

Analyze and track your performance with our Testing and Analytics Package.